RegistrierenRegistrieren   LoginLogin   FAQFAQ    SuchenSuchen   
Strömung in einem geschlossenem System
 
Neue Frage »
Antworten »
    Foren-Übersicht -> Mechanik
Autor Nachricht
dontknow22



Anmeldungsdatum: 07.02.2008
Beiträge: 6

Beitrag dontknow22 Verfasst am: 07. Feb 2008 21:42    Titel: Strömung in einem geschlossenem System Antworten mit Zitat

Hallo, ich bräuchte die formel für das folgende Problem:

Ein Rohr (rund)mit einer Steigung... Wasser soll durch dieses rohr mit druck durchgepumpt werden. nun muss ich die fließgeschwindigkeit unter berücksichtigung des druckabfall berechnen

dabei spielt die reibung keine rolle.

dieses modell soll ich danach mit scilab modellieren

vielen dank für eure hilfe
dermarkus
Administrator


Anmeldungsdatum: 12.01.2006
Beiträge: 14788

Beitrag dermarkus Verfasst am: 07. Feb 2008 21:56    Titel: Antworten mit Zitat

Könnte es sein, dass du da gerade den Aufgabenteil weggelassen hast, in dem die Informationen drinstehen, die du hier am besten gebrauchen kannst?

Was würdest du sagen, wodurch kommt hier der Druckabfall im Rohr zustande?
dontknow22



Anmeldungsdatum: 07.02.2008
Beiträge: 6

Beitrag dontknow22 Verfasst am: 07. Feb 2008 21:59    Titel: Antworten mit Zitat

durch die erdanziehung verringert sich ja die fließgeschwindigkeit in dem modell... und ich bräuchte eine formel um zu errechnen. also die verringerung der fließgeschwindikkeit und ich weiß nicht, ob sich druck auch dadurch verringert..??
dermarkus
Administrator


Anmeldungsdatum: 12.01.2006
Beiträge: 14788

Beitrag dermarkus Verfasst am: 07. Feb 2008 22:04    Titel: Antworten mit Zitat

Welche Formeln hast du denn bisher selbst schon zu Flüssigkeiten gelernt, die durch ein Rohr fließen? Ist da auch eine dabei, in der die Erdanziehung und die Fließgeschwindigkeit vorkommt? Kennst du so eine Gleichung unter Umständen unter dem Namen Bernoulli-Gleichung?
dontknow22



Anmeldungsdatum: 07.02.2008
Beiträge: 6

Beitrag dontknow22 Verfasst am: 07. Feb 2008 22:23    Titel: Antworten mit Zitat

ja aber wie kann ich mit der die fließgeschwindigkeit berechnen und wie spielt da dei steigung mit rein...
tut mir leid ich habe absolut keine ahnung....
dermarkus
Administrator


Anmeldungsdatum: 12.01.2006
Beiträge: 14788

Beitrag dermarkus Verfasst am: 07. Feb 2008 22:34    Titel: Antworten mit Zitat

Schreib doch erst mal das hin, was du schon weißt und/oder schon gefunden hast.

Hast du schon eine Form der Bernoulli-Gleichung gefunden, die dir hier nützlich vorkommt? Dann schreibe sie am besten hier auf smile

Und magst du wirklich mal den Teil deiner Aufgabenstellung noch mit hier angeben, in dem es um die Steigung geht, die du hier früher oder später brauchen wirst?
dontknow22



Anmeldungsdatum: 07.02.2008
Beiträge: 6

Beitrag dontknow22 Verfasst am: 07. Feb 2008 23:03    Titel: Antworten mit Zitat

Das Modell besteht aus einem geschlossenen Rohrsystem, in dem Wasser entweder ungehindert hindurch fließen kann oder auf Grund von Widerständen behindert wird. Dabei untersuchen wir den Zusammenhang zwischen dem Druck und der Fließgeschwindigkeit in einem waagerecht liegenden Rohr mit Widerstand und einem geneigtem Rohr ohne Widerstand. Solche Widerstände können festsitzende Gegenstände in einem Rohr sein oder auch Verengungen an einer Stelle des Rohres, die wiederum die Fließgeschwindigkeit und den Druck verändern. Diese beiden Komponenten werden an jeder Stelle des Rohres berechnet und graphisch dargestellt, um das Verhältnis der beiden Hauptkomponenten Druck und Fließgeschwindigkeit zu verdeutlichen. Des Weiteren werden diverse Annahmen aufgestellt, um das Modell zu vereinfachen. In einem solchen Modell müssen also folgende mathematische und physikalische Aspekte berücksichtigt werden:
- die Fließgeschwindigkeit,
- der Druck,
- die Größe (Durchmesser) des Rohres,
- der Grad der Steigung,
- die Dichte des Wassers,
- die Form des Widerstandes.


dazu die formeln

g*z+p/rho+v²/2=const.
1/2 rho v²+rho g h+p=const.

das ist das was ich weiß
dontknow22



Anmeldungsdatum: 07.02.2008
Beiträge: 6

Beitrag dontknow22 Verfasst am: 07. Feb 2008 23:04    Titel: Antworten mit Zitat

also ich habe den teil mit der steigung und nicht mit dem widerstand...
dermarkus
Administrator


Anmeldungsdatum: 12.01.2006
Beiträge: 14788

Beitrag dermarkus Verfasst am: 07. Feb 2008 23:15    Titel: Antworten mit Zitat

Danke, damit verstehe ich schon deutlich besser, worum es geht smile

Wo in deinen Gleichungen findest du nun eine Eigenschaft deiner Steigung wieder? In welcher Form steht diese Steigung in deinen Formeln drin?

Hast du in deinem Teil auch Rohrverengungen oder nicht? Was folgerst du daraus für die Fließgeschwindigkeit in deinem Teil des geschlossenen Rohrsystems?
derIng



Anmeldungsdatum: 03.02.2008
Beiträge: 51

Beitrag derIng Verfasst am: 08. Feb 2008 08:26    Titel: Antworten mit Zitat

Es gibt für die Aufgabe sicher ein Bild. Wenn nicht, würde ich empfehlen, eine Skizze für das Problem zu entwerfen.
In der Skizze kann man dann die Lage des/der Rohre, die Richtung der Schwerkraft, die Punkte, an welchen z.B. der Druck oder die Fließgeschwindigkeit bekannt ist eintragen. Gleichen Größen (z.B. Drücken) an verschiedenen Stellen gibt man dann verschiedene Indizes.

Und schon kann man ganz entspannt die Formeln hinschreiben und ist sich sicher über etwa das gleiche zu diskutieren.

Ich sehe, man kann Anhange anfügen. Ein BMP, GIG, JPG oder PDF wäre da sicher geeignet.
dontknow22



Anmeldungsdatum: 07.02.2008
Beiträge: 6

Beitrag dontknow22 Verfasst am: 08. Feb 2008 09:22    Titel: Antworten mit Zitat

so hier einmal meine annahmen und ein bild


Neu OpenDocument Text.pdf
 Beschreibung:

Download
 Dateiname:  Neu OpenDocument Text.pdf
 Dateigröße:  42.08 KB
 Heruntergeladen:  196 mal

dermarkus
Administrator


Anmeldungsdatum: 12.01.2006
Beiträge: 14788

Beitrag dermarkus Verfasst am: 08. Feb 2008 12:26    Titel: Antworten mit Zitat

Das Bild ist schon mal prima, damit kannst du bestimmt gut arbeiten smile

Magst du mal deine letzten beiden Annahmen genau anschauen und überprüfen, ob es wirklich möglich ist, beide gleichzeitig zu erfüllen?

Also:

"Der Volumenstrom beträgt 200 m^3/h"
(hast du diese Information fertig vom Aufgabensteller bekommen?)
und
"Die Fließgeschwindigkeit nimmt proportional zum Druckabfall ab".

Gehe ich recht in der Annahme, wenn ich deiner Skizze entnehme, dass in deinem Teil des Rohrsystems der Rohrquerschnitt konstant bleibt und in diesem Teil weder Wasser durch abzweigende Abflüsse verloren geht noch Wasser durch Zuflüsse zum Strom im Hauptrohr hinzugefügt wird? Hilft dir das bei deinen Überlegungen?
Neue Frage »
Antworten »
    Foren-Übersicht -> Mechanik